Search found 11 matches


Hi Rarns,

When 1 is divided by 4, the remainder is 1 and quotient is 0.

Think you've done a silly mistake.

Sid

by sidvenki

Thu Jun 16, 2011 5:59 pm
Forum: Data Sufficiency
Topic: Odd/Even
Replies: 2
Views: 1030

The second bold face is not a conclusion, but a claim to aids the conclusion. Conclusion : Nevertheless, these facts do not warrant the conclusion drawn by some commentators that in the countries with the higher rates of reported whiplash injuries, half of the reported cases are spurious. So Options...

by sidvenki

Mon Jun 13, 2011 6:45 pm
Forum: Critical Reasoning
Topic: boldface question
Replies: 3
Views: 1316

Option E does not weaken the argument. Rather it is giving a reason as to why there was a decline in the quality of the carpenters, thereby supporting the claim.

by sidvenki

Sun Jun 12, 2011 11:39 pm
Forum: Critical Reasoning
Topic: Need Help
Replies: 5
Views: 1514

IMO: A

As it implies that tropical people was exposed to aflatoxin.


Sid

by sidvenki

Sun Jun 12, 2011 9:58 pm
Forum: Critical Reasoning
Topic: A certain tropical --- a nice tricky CR --- reposting
Replies: 6
Views: 4880

Option A has the pronoun their. The subject is singular "The Turtle".

B,C,D has problems in the end. for teeth, as do teeth and as teeth do respectively.


Option E is therefore the correct answer.

Sid

by sidvenki

Thu Jun 09, 2011 6:52 pm
Forum: Sentence Correction
Topic: Although the turtle has been toothless
Replies: 5
Views: 4282

If Pelvis, then hind limbs.

Answer is A.

Option B almost weakens the argument.
Option C uncertainty among scientists in this theory.
Option D, E are totally out of scope, as it talks about other sea animals.


Sid

by sidvenki

Mon May 30, 2011 4:25 pm
Forum: Critical Reasoning
Topic: A good must be true question!!!!
Replies: 3
Views: 1600

In question 3, when paleobotanists discover that ancient forests died away due to reduced sunlight, it only supports the claim that there was lesser sunlight, therefore eradication of plants. In option A, it goes beyond this and tries to fit in the reason for this lesser sunlight. This lesser sunlig...

by sidvenki

Thu May 26, 2011 6:01 pm
Forum: Reading Comprehension
Topic: Good RC-99, passage-44
Replies: 5
Views: 2148

Option C is incorrect as increasing the actual flying time may not necessarily result in decrease in the Air Crashes. Reason is because, lets assume that minimum 100hrs (flying time) is required to prevent crashes. Initially lets assume that the hours of flying was only 40. Increasing it to 80 will ...

by sidvenki

Thu May 26, 2011 1:13 am
Forum: Critical Reasoning
Topic: Airline crashes
Replies: 9
Views: 2431

Hi All, Rather than CHOOSING A, we can eliminate all the others very easily. 1) w^4 and w^2 are divisible by w itself. 2) w/4, If w is a prime no, then w/4 will definitely be a fraction. So not a prime number . 3) 5w is divisible by 5. All the other 4 options definitely will not give a prime number....

by sidvenki

Thu May 19, 2011 4:53 pm
Forum: Problem Solving
Topic: problem with prime numbers
Replies: 8
Views: 1832

Pronoun in a clause before modifying its noun

Hi All, I have a doubt! Is the following sentence formation correct? Reaching the zenith of its sales in the 1960s, the spy novel... Is this correct? I feel that one cannot use a pronoun without any reference and in the above its refers to spy novel. But I did find the same sentence structure in som...

by sidvenki

Wed Mar 16, 2011 7:08 am
Forum: Sentence Correction
Topic: Pronoun in a clause before modifying its noun
Replies: 1
Views: 944

2nd is a lot easier than the 1st one. x is factor of y => y = k1*x y is a factor of z => z = k2*y 1) Z is even if y is even. so statement B is sufficient to answer z is even. 2) Statement A is xz = even; z = k2y = k1k2x; Given as xz is even; either x is even or z is even or both. Assuming x is even-...

by sidvenki

Wed Oct 20, 2010 6:46 pm
Forum: GMAT Math
Topic: Please Help---Quantative Problems
Replies: 2
Views: 1479